Search found 83 matches


Averages Problem

Q. x, y, and z are consecutive integers, and x < y < z. What is the average of x, y, and z?
(1) x = 11
(2) The average of y and z is 12.5.

IMO - D, am I correct?

by MI3

Tue Jul 19, 2011 11:59 am
Forum: Data Sufficiency
Topic: Averages Problem
Replies: 1
Views: 885

Averages

Q. L spends total $6.00 for one kind of D and one kind of C. How many D did he buy?
(1) the price of 2D was $0.10 less than the price of 3C
(2) the average price of 1 D and 1 C was $0.35

Unfortunately, I don't have the resolution to the above problem, please opine.

Thanks,
M

by MI3

Tue Jul 19, 2011 11:58 am
Forum: Data Sufficiency
Topic: Averages
Replies: 0
Views: 818

Combined) This is where it will serve you to know the weighted average formula well. Combined, the statements tell us that the manager average is $5,000 below the weighted average, and that the directors average is $15,000 above. With A = weighted average salary of all employees [M(A - 5000) + D(A ...

by MI3

Tue Jul 19, 2011 11:47 am
Forum: Data Sufficiency
Topic: Percentages
Replies: 3
Views: 1170

IMO the choice is A Statement (1): N is prime and N^6 is even hence N is even as well. So N will be 2 (as 2 is the only even prime number) Hence (1) is sufficient. Note: Prime numbers can only be positive. Statement (2): 2N+14<20 and hence N<3. But we cant guess the exact value of N. Hence (2) is n...

by MI3

Sun Jul 17, 2011 7:56 am
Forum: Data Sufficiency
Topic: Median
Replies: 11
Views: 2429

We are given 4 of the 5 numbers in the set {-9, -8, 4, 15, N} with N as an unknown value. If -8 <= N <= 4, then N is the median, since there are an odd number of integers in the set. If N < -8 or N > 4, then (-8 + 4)/2 = -2 is the median. In other words, if we know that -8 <= N <= 4, we must know i...

by MI3

Sun Jul 17, 2011 7:51 am
Forum: Data Sufficiency
Topic: Median
Replies: 11
Views: 2429

Q. Set X consists of different positive numbers arranged in ascending order: K, L, M, 5, 7. If K, L and M are consecutive integers, what is the arithmetic mean of set X? (1) The product K × L × M is a multiple of 6 (2) There are at least 2 prime numbers among K, L and M IMO - A , am I correct? Hi...

by MI3

Sun Jul 17, 2011 7:42 am
Forum: Data Sufficiency
Topic: Arithmetic mean
Replies: 3
Views: 2406

Median

Q. What is the median of set A {-8, 15, -9, 4, N}?
(1) N is a prime and N^6 is even (2) 2N + 14 < 20

Is the answer E ?

by MI3

Sun Jul 17, 2011 7:28 am
Forum: Data Sufficiency
Topic: Median
Replies: 11
Views: 2429

MI3 wrote:Q. Which of the following could be the median of a set consisting of 6 different primes?
(A) 2 (B) 3 (C) 9.5 (D) 12.5 (E) 39
Sorry - Ignore the above question, I figured it out, answer is E.

by MI3

Sun Jul 17, 2011 7:12 am
Forum: Problem Solving
Topic: Median
Replies: 3
Views: 3189

Median

Q. Which of the following could be the median of a set consisting of 6 different primes?
(A) 2 (B) 3 (C) 9.5 (D) 12.5 (E) 39

by MI3

Sun Jul 17, 2011 7:08 am
Forum: Problem Solving
Topic: Median
Replies: 3
Views: 3189

Credit card problem!

Q. On Jane's credit card account, the average daily balance for a 30-day billing cycle is average (arithmetic mean) of the daily balances at the end of the 30 days. At the beginning of a certain 30-day billing cycle, Jane's credit card account had a balance of $600. Jane made a payment of $300 on th...

by MI3

Sun Jul 17, 2011 3:11 am
Forum: Data Sufficiency
Topic: Credit card problem!
Replies: 2
Views: 1110

DS Question

Q. A new cell phone plan is offering pricing based on average monthly use. Brandon and Jodie are comparing their average use to determine the best plan for them. Brandon's average monthly usage in 2001 was q minutes. Was this less than, greater than, or equal to Jodie's 2001 average monthly usage, i...

by MI3

Sun Jul 17, 2011 3:01 am
Forum: Data Sufficiency
Topic: DS Question
Replies: 1
Views: 1184

Average score

Q. The average score of x number of exams is y. When an additional exam of score z is added in, does the average score of the exams increase by 50%?
(1) 3x = y (2) 2z - 3y = xy

Please opine on how to resolve the above problem.

by MI3

Sun Jul 17, 2011 2:51 am
Forum: Data Sufficiency
Topic: Average score
Replies: 3
Views: 1821

Mean - Question

Q. If numbers N and K are added to set X {2, 8, 10, 12}, its mean will increase by 25%. What is the value of N^2 + 2NK + K^2 ?
(A) 28 (B) 32 (C) 64 (D) 784 (E) 3600

IMO D, am I correct?

by MI3

Sun Jul 17, 2011 2:48 am
Forum: Problem Solving
Topic: Mean - Question
Replies: 2
Views: 1129

Arithmetic mean

Q. Set X consists of different positive numbers arranged in ascending order: K, L, M, 5, 7. If K, L and M are consecutive integers, what is the arithmetic mean of set X?
(1) The product K × L × M is a multiple of 6
(2) There are at least 2 prime numbers among K, L and M

IMO - A, am I correct?

by MI3

Sun Jul 17, 2011 2:46 am
Forum: Data Sufficiency
Topic: Arithmetic mean
Replies: 3
Views: 2406

Integers

Q. If set R contains the consecutive integers from -5 to -1, what is the mean of set R?
A. -5 B. -3 C. 0 D. 3 E. 5

IMO - B, am I correct?

by MI3

Sun Jul 17, 2011 2:45 am
Forum: Problem Solving
Topic: Integers
Replies: 1
Views: 915